Difference between revisions of "2024 SSMO Team Round Problems/Problem 8"
(Created page with "==Problem== Three integers <math>0\le a\le b\le c< 229</math> satisfy the congruence <math>n^3 \equiv 1 \pmod{229}.</math> Given that <math>71^2 - 3</math> and <math>107^2 +...") |
|||
Line 4: | Line 4: | ||
==Solution== | ==Solution== | ||
+ | Note that <math>n^3\equiv1\pmod{229}\implies (n^3-1)\equiv0\pmod{229}.</math> Consider the complex third roots of unity <math>1</math> and <math>\frac{-1\pm i\sqrt{3}}{2}.</math> They can be defined based on <math>\frac12,\sqrt{-1},</math> and <math>\sqrt3.</math> From the given multiples of <math>229,</math> we can <math>\sqrt{3}\equiv71\pmod{229}</math> and <math>i\equiv107\pmod{229}</math> since they function like square roots. In addition, <math>229</math> is odd so <math>2^{-1}</math> exists. We have <math>i\sqrt{3}\equiv107\cdot71\equiv269\pmod{229}.</math> Thus, <math>\frac{-1+i\sqrt{3}}{2} \equiv 134\pmod{229}</math> and <math>\frac{-1-i\sqrt{3}}{2}\equiv94\pmod{229}.</math> So, <math>a = 1,b = \boxed{94},</math> and <math>c = 134.</math> | ||
+ | |||
+ | ~SMO_Team |
Latest revision as of 14:38, 10 September 2025
Problem
Three integers satisfy the congruence
Given that
and
are both multiples of
find the value of
Solution
Note that Consider the complex third roots of unity
and
They can be defined based on
and
From the given multiples of
we can
and
since they function like square roots. In addition,
is odd so
exists. We have
Thus,
and
So,
and
~SMO_Team